Recherche sur le blog!

Concours Physique ENSI 1993 (Énoncé)

Étude d'un cyclotron dans l'approximation non relativiste, diffusion p‑p.
Ce problème est à traiter intégralement dans l'approximation Non Relativiste. I ‑ Un cyclotron accélérant des protons est constitué d'un électro‑aimant à pôles plans dans l'entrefer duquel règne un vide poussé (figure 1). La région de l'espace contenant les trajectoires des protons, ou volume d'accélération, est un cylindre dont les faces sont parallèles aux plans des pôles. Le champ magnétique B0 y est uniforme, permanent et perpendiculaire aux faces planes du cylindre. Les parois du cylindre sont matérialisées par deux électrodes conductrices creuses, en cuivre, appelées dés. Ces dés sont séparés par une région d'épaisseur g faible, s'étendant de part et d'autre d'un plan contenant l'axe du cylindre. Les faces en regard des dés sont parallèles ; l'intersection de l'axe du cylindre avec le plan médian des dés est considérée comme le centre du cyclotron. Un dispositif, appelé source, produit des protons qui sont injectés au centre avec une énergie cinétique négligeable. Un générateur permet d'appliquer entre les dés une tension alternative à haute fréquence ${u_c} = {U_c}\sin \left( {\omega t + \Phi } \right)$créant entre les dés un champ électrique uniforme $\vec \varepsilon $ tel que ${u_c} = \varepsilon g$.

Dans les dés les protons décrivent donc des trajectoires qui sont successivement des demi cercles (figure 1). La fréquence ${\nu _c} = \frac{\omega }{{2\pi }}$du champ électrique $\vec \varepsilon $ est choisie de telle sorte que, pendant qu'un proton décrit sensiblement un demi cercle dans un dé, la phase du champ électrique augmente de π. Ainsi, les protons ayant la phase convenable sont accélérés à chaque passage dans l'espace inter dés. On admettra que les protons sont accélérés une première fois sur la distance g avant de décrire le premier demi cercle.
1) Sachant que la masse du proton est mp, sa charge e, calculer la fréquence ${\nu _c}$du champ électrique $\vec \varepsilon $.
A.N. : mp = 1,6.10 ‑ 27 kg, e = 1,6.10 -19 Coulomb, Bo = 1,5 Tesla
2) En négligeant l'épaisseur g de l'espace accélérateur (entre dés) calculer le rayon rn du nième demi cercle décrit par les protons. On suppose qu'à la sortie de la source, ils traversent l'espace accélérateur lorsque ${u_c} = {U_c}$
A.N. : Uc = 50 kV, n = 400
3) En réalité les protons traversent le plan médiateur de l'espace accélérateur à un instant tc tel que $\omega .{t_c} = 0$à Kπ près et avec une phase Φ0. Cet espace a une largeur g de l'ordre de 1 cm et il est nécessaire de tenir compte au cours de sa traversée de la variation du champ accélérateur (figure 1).
a) Montrer que le gain en énergie pour une orbite située à une distance r du centre du cyclotron est $w = e\,\,{U_c}\left[ {\frac{{\sin \left( {g/2r} \right)}}{{\left( {g/2r} \right)}}} \right]\sin {\Phi _0}$ en admettant que dans l'espace accélérateur ($ - \frac{g}{2} \le x \le \frac{g}{2}$) on peut écrire x = v t ou v est la vitesse des protons (v = ωr) sensiblement constante dans l'intervalle g, (g < r).
b) Quelle valeur s'efforcera‑t‑on d'obtenir pour Φ0 ?
c) En faisant les approximations convenables, calculer l'énergie cinétique W obtenue à la sortie du cyclotron, c'est‑à‑dire après la nième demi orbite avec la valeur de Φ0 obtenue en 3)b. On l'exprimera en joules et en MeV (Mega électron Volts). On montrera que l'approximation non relativiste est acceptable.
d) Les protons arrivent à la sortie par paquets séparés les uns des autres par le même intervalle de temps, alors que l'injection se fait de façon continue au centre de l'accélérateur.
Expliquer l'origine de ces paquets.
Calculer l'intervalle de temps séparant deux paquets de protons.
A.N. : Uc = 50 kV, n = 400

4) En pratique, les protons sont injectés dans le cyclotron avec une phase comprise dans l'intervalle${\Phi _0} \pm \Delta \Phi $ avec $\Delta \Phi = \pm 1,5^\circ $. Il en résulte une fluctuation en énergie ΔW.
a) Calculer la résolution en énergie $\frac{{\Delta W}}{W}$du faisceau sortant pour n fixé.
b) En déduire la variation relative de rayon $\frac{{\Delta {r_n}}}{{{r_n}}}$qui en résulte ainsi que$\Delta {r_n}$.
A.N. : Uc = 50 kV, n = 400
5) Pour extraire les protons du cyclotron, on les fait passer dans un déflecteur électrostatique qui a pour but de permettre au faisceau de sortir de la zone où règne un champ magnétique. Ce déflecteur est constitué de 2 lames métalliques courbées et parallèles entre lesquelles on applique une différence de potentiel constante UE afin de créer un champ électrique $\overrightarrow {{E_E}} $ constamment perpendiculaire à la vitesse de la particule, au champ magnétique et dirigé vers l'extérieur (figure 1bis). On se bornera à examiner les conditions d'entrée des particules dans le déflecteur.
a) Pour que les protons correspondant au (n ‑ 2)ième demi cercle ne pénètrent pas entre les 2 lames et que ceux du nième demi cercle y pénètrent, on place la lame intérieure du déflecteur à un rayon r tel qu'elle soit à égale distance du (n ‑ 2)ième et du nième demi cercle. Quelle est la valeur de r pour n = 400 ? On admettra que pour n = 400 les demi cercles ont pour centre le centre du cyclotron.
b) Montrer que l'on peut tolérer des valeurs de l' "acceptance de phase" $\Delta {\Phi _r}$plus grandes que la valeur $\Delta \Phi = 1,5^\circ $prise dans la question précédente. Calculer la valeur maximum $\Delta {\Phi _r}$ admissible pour que tous les protons de la nième orbite pénètrent dans le déflecteur sans que ceux de la (n ‑ 2)ième n'y pénètrent.
A.N. : n = 400 6) Les protons sortant de la source ont une quantité de mouvement faible qui a une composante le long de l'axe du cyclotron dont la valeur maximum est ± mva (va étant la vitesse axiale des protons). Sachant que les protons ne subissent pas d'accélération le long de l'axe jusqu'à ce qu'ils pénètrent dans le déflecteur et sachant que la hauteur disponible par rapport au plan médian dans lequel se trouve la source est h = ± 5 cm, calculer l'énergie cinétique axiale acceptable au départ de la source afin qu'ils ne heurtent pas les parois supérieure ou inférieure des dés. On notera cette énergie par Wa et on l'exprimera en électrons volts. Quelles remarques vous inspire cette valeur ?

II ‑ Le cyclotron qui ne fonctionne pas à l'énergie maximum, calculée précédemment, fournit un faisceau de protons d'énergie cinétique T = 15 MeV que l'on envoie sur une cible d'aluminium dans laquelle on se propose de mettre en évidence des impuretés d'hydrogène. Pour ceci, on étudie la diffusion élastique proton‑proton que l'on compare à la diffusion élastique proton‑ aluminium. On dispose un détecteur de protons D1 selon une direction faisant un angle de 30° avec la direction des protons incidents (figure 2). Pour être bien certain d'avoir une diffusion proton‑proton, on dispose un deuxième détecteur D2 à un angle ψ dans le même plan de diffusion et l'on identifiera comme diffusion proton‑proton la détection simultanée dans les deux détecteurs Dl et D2 des deux protons pl et p2.
1) Calculer à quel angle ψ on doit placer ce deuxième détecteur.
2) Calculer l'énergie cinétique des protons pl (30°) et p2 (ψ). 3) Lorsque la diffusion du proton a lieu sur un noyau d'aluminium dont la masse est 27 fois celle du proton, calculer à quel angle α sera diffusé le noyau d'aluminium, le proton étant toujours diffusé à 30°; sera‑t‑il facile de distinguer une diffusion proton‑proton d'une diffusion proton‑aluminium ?

Ill ‑ Le cyclotron fournissant toujours des protons de 15 MeV, on se propose de les
post-­accélérer en les injectant dans un accélérateur linéaire constitué d'une série de conducteurs métalliques creux, ayant la symétrie de révolution et de même axe appelés cavités. Les protons sont injectés le long de l'axe de cet accélérateur. Les cavités sont connectées comme il est indiqué sur le schéma figure 3. La tension du générateur vaut ${u_L} = {U_L}\sin \left( {2\pi {\nu _L}t} \right)$
Les protons de 15 MeV pénètrent dans la première cavité lorsque uL = 0 c'est‑à‑dire à t=0. Ils doivent ensuite être accélérés par une tension uL = UL toutes les fois qu'ils passent d'une cavité à la suivante.
1) Si ${\nu _c}$est la fréquence de la tension accélératrice du cyclotron, quelle devra être la fréquence la plus basse ${\nu _L}$de l'accélérateur linéaire pour accélérer tous les protons issus du cyclotron.
2) A chaque passage entre 2 cavités les protons sont accélérés et leur énergie augmente de eUL. Calculer la longueur minimum de la première et de la (n ‑ 1)ième cavité.
A. N. UL = 100 kV n = 20
Dans cette question, on considérera que la longueur de l'espace entre cavités est négligeable par rapport à la longueur des cavités.
3) En toute rigueur, pour conserver une bonne stabilité du faisceau, le franchissement des espaces accélérateurs s'effectue lorsque uL est une fonction croissante du temps. Soit ΦS la différence de phase par rapport à celle correspondant au gain maximum en énergie pour laquelle $\sin \left( {2\pi {\nu _L}t} \right) = 1$
Si ${\Phi _S} = 5^\circ $ quelle sera la nouvelle valeur à donner à UL pour que cet accélérateur linéaire conserve les caractéristiques définies plus haut.

Concours Physique ESEM (Spéciale C) 1993 (Corrigé)

ESEM Orléans 1993 ELECTROPHORESE
I Modélisation de la colonne poreuse
I.1. La résistance d'un conducteur est R= U/I = (1/γ')l/S
Donc ici $\gamma '\, = \frac{I}{{{U_1} - {U_2}}}\frac{{l'}}{{S'}}$ soit numériquement γ'=0.83 Ω-1 .m-1
I.2 .$S\ell \, = S'\ell '\, - (M/\rho )$ ( = volume total – volume des fibres )

$R = \frac{1}{\gamma }\frac{\ell }{S} = \frac{1}{{\gamma '}}\frac{{\ell '}}{{S'}}$ => $\ell = \ell '\frac{\gamma }{{\gamma '}}\frac{S}{{S'}}$ et de même $S = \frac{\ell }{{\ell '}}\frac{{\gamma '}}{\gamma }S'$
en remplaçant dans l'expression précédemment obtenue :
$S = (S'\ell '\, - (M/\rho )\frac{1}{\ell }$ => $S = \sqrt {(S'\ell '\, - (M/\rho )\frac{1}{{\ell '}}'\frac{{\gamma '}}{\gamma }S'} $ et $\ell = \sqrt {(S'\ell '\, - (M/\rho )\frac{1}{{S'}}'\frac{\gamma }{{\gamma '}}S} $
S= 1.7 cm2 et l= 31 cm
II Etude du mouvement d'un ion
II.1. $\vec E\, = \,\frac{{{U_1} - {U_2}}}{\ell }{\vec u_x}$ et $\vec F\, = \,q\vec E$
II.2. $m\frac{{d\vec v}}{{dt}} = q\vec E\, - f\vec v$
s'intègre en $\vec v = \,{\vec v_0}\,{e^{ - \frac{{f\,t}}{m}}} + \frac{q}{m}\vec E$ et à t= 0 v= 0 => $\vec v = \,\frac{q}{f}\vec E(1 - {e^{ - \frac{{f\,t}}{m}}})$
La vitesse limite ${\vec v_\infty } = \,\frac{q}{f}\vec E$est atteinte à 5% près pour t1 tel que $v({t_1}) = 0.95{v_\infty } = > \,(1 - {e^{ - \frac{{f\,t}}{m}}}) = 0.95\,\, = > \frac{{f\,{t_1}}}{m} = - \ln \,0.05\,\, = > $${t_1}\, = \,\frac{m}{f}\ln \,20$
${\vec v_\infty } = \,\frac{q}{f}\vec E = \mu \vec E$ => $f = \,\frac{q}{\mu }$ ${t_1}\, = \,\frac{{m\mu }}{q}\ln \,20\,\, = \,\,\,{9.410^{ - 14}}s$ et v ≅2 10-5s

On peut donc considérer que cette vitesse limite est atteinte quasi-instantanément.
La durée possible de l'expérience est la durée de parcours des ions du centre vers un bout e de la cuve Soit τ ≅ l/2v≅ 2 heures
III Etude de la diffusion
III.1. La répartition des molécules est homogène selon y et z , et il n'y a pas de diffusion dans ces directions
III.2. j en m-2s-1 et D en m2s-1
III.3. le nombre de particules à l’intérieur du cylindre dS dx varie de
dN = jn(x) S dt – jn(x+dx)S dt $dN = \, - \,\frac{{\partial {j_n}}}{{\partial x}}\,dx\,S\,dt\,$
La concentration particulaire = nb de particule par unité
de volume=n=(N/S)dx varie donc de dn = dN / S dx $dn = \, - \,\frac{{\partial {j_n}}}{{\partial x}}\,\,dt\,$donc $\frac{{\partial n}}{{\partial t}} = \, - \,\frac{{\partial {j_n}}}{{\partial x}}\,\,\,$
III.4. Loi de Fick ${j_{}}\, = \, - D\,\frac{{\partial n}}{{\partial x}}$ et Bilan de particules $\frac{{\partial n}}{{\partial t}} = \, - \,\frac{{\partial j}}{{\partial x}}\,\,\,$=>$\frac{{\partial n}}{{\partial t}} = \,D\,\frac{{{\partial ^2}n}}{{\partial {x^2}}}\,\,\,$
III.5. $n(x,t) = \frac{A}{{\sqrt t }}{e^{ - B\frac{{{x^2}}}{t}}}$ =>
$\frac{{\partial n}}{{\partial t}} = A[\frac{{ - 1}}{2}{t^{ - 3/2}}{e^{ - B\frac{{{x^2}}}{t}}} + {t^{ - 1/2}}(\frac{{B{x^2}}}{{{t^2}}}{e^{ - B\frac{{{x^2}}}{t}}})] = \frac{A}{{2{t^{3/2}}}}{e^{ - B\frac{{{x^2}}}{t}}}( - 1 + \frac{{2B{x^2}}}{t})$
$\frac{{\partial n}}{{\partial x}} = \frac{A}{{\sqrt t }}( - \frac{B}{t}2x{e^{ - B\frac{{{x^2}}}{t}}})$et $\frac{{{\partial ^2}n}}{{\partial {x^2}}} = \frac{A}{{\sqrt t }}( - \frac{B}{t}2)[{e^{ - B\frac{{{x^2}}}{t}}} + x( - \frac{B}{t}2x{e^{ - B\frac{{{x^2}}}{t}}})] = \frac{{2AB}}{{{t^{3/2}}}}{e^{ - B\frac{{{x^2}}}{t}}}[ - 1 + \frac{{2B{x^2}}}{t}]$
$\frac{{{\partial ^2}n}}{{\partial {x^2}}} = 4B[\frac{A}{{{t^{3/2}}}}{e^{ - B\frac{{{x^2}}}{t}}}[ - 1 + \frac{{2B{x^2}}}{t}]] = 4B\,\frac{{\partial n}}{{\partial t}}$
Ces deux dérivées vérifient bien d'équation de diffusion avec D = 1/4B

III.6.

III.7. A la date t, 95 % des molécules sont dans la zone de largeur Δl, si la probabilité pour que |x| >Δl/2 est égale à 5%
Or, n(x,t) suit une loi de Gauss , pour laquelle on nous rappelle que cette probabilité de 5% correspond à "σ " soit ici 2σ2=t/B= 4Dt => Donc $\Delta \ell = 2\sqrt {2Dt} $
IV Etude du phénomène général
IV.1.

IV.2.La séparation est convenable si

${v_1}t + \sqrt {2{D_1}t} \,\,\,\, < {v_2}t - \sqrt {2{D_2}t} $
soit $({v_2} - {v_1})t > (\sqrt {2{D_1}} + \sqrt {2{D_2}} )\sqrt t $
$t\,\,\, > {\left[ {\frac{{\sqrt {2{D_1}} + \sqrt {2{D_2}} }}{{{v_2} - {v_1}}}} \right]^2}$
t= 41 s

Concours Physique TPE 1992 (Corrigé)

  1. Sur le système (O,$\vec u,\vec v$), P a pour coordonnées (a,0) et Ω a pour coordonnées (-a cos θ, a sin θ) ; d’où : $P\vec \Omega = - a\left( {1 + \cos \theta } \right)\vec u + a\sin \theta \vec v$
  2. $P\Omega = 2a\cos \frac{\theta }{2}$ s’obtient à partir des coordonnées précédentes ou en raisonnant directement sur le triangle isocèle OPΩ dont les angles P et Ω valent θ/2.
  3. $\vec T = K\left( {2a\cos \frac{\theta }{2} - \ell } \right)\left[ { - \cos \frac{\theta }{2}\vec u + \sin \frac{\theta }{2}\vec v} \right]$
  4. $\vec F = Mg\left[ {\cos \theta \vec u - \sin \theta \vec v} \right] + r\vec u + K\left( {2a\cos \frac{\theta }{2} - \ell } \right)\left[ { - \cos \frac{\theta }{2}\vec u + \sin \frac{\theta }{2}\vec v} \right]$
    où r est la mesure algébrique (et non le module) de la réaction du demi cercle sur la perle.

  5. $\vec V = a\dot \theta \vec v$.
  6. $\vec F.\vec V = a\dot \theta \left[ {\left( {Ka - Mg} \right)\sin \theta - K\ell \sin \frac{\theta }{2}} \right]$.
  7. $\vec F.\vec V = - \frac{{d{E_p}}}{{dt}} = - \frac{{d{E_p}}}{{d\theta }}\frac{{d\theta }}{{dt}}$. D’où $\frac{{d{E_p}}}{{d\theta }} = a\left[ {\left( {Mg - Ka} \right)\sin \theta + K\ell \sin \frac{\theta }{2}} \right]$ et
    ${E_p} = a\left[ {\left( {Ka - Mg} \right)\cos \theta - 2K\ell \cos \frac{\theta }{2}} \right] + cste$
  8. $E = \frac{1}{2}M{a^2}{\dot \theta ^2} + {E_p}$
  9. En projetant la loi fondamentale de la dynamique sur $\vec v$ et en utilisant l’expression de la force de la question 4, on obtient : $Ma\ddot \theta = {F_\theta } = - Mg\sin \theta + K\left( {2a\cos \frac{\theta }{2} - \ell } \right)\sin \frac{\theta }{2} = \left( {Ka - Mg} \right)\sin \theta - K\ell \sin \frac{\theta }{2}$
    On peut aussi obtenir cette relation en dérivant l’énergie par rapport au temps.
  10. Les positions d’équilibres sont celles pour lesquelles: ${F_\theta } = 2\left( {Ka - Mg} \right)\sin \frac{\theta }{2}\cos \frac{\theta }{2} - K\ell \sin \frac{\theta }{2} = 0$
    - ou bien $\theta = 0$ ;
    - ou bien $\theta = \pm \,{\theta _1}$ tel que $\cos \frac{{{\theta _1}}}{2} = \frac{{K\ell }}{{2\left( {Ka - Mg} \right)}}$ si cette équation a des racines.
  11. ${\theta _1}$, compris entre 0 et $\frac{\pi }{2}$(soit $\frac{{{\theta _1}}}{2}$ compris entre 0 et $\frac{\pi }{4}$), existe si $\frac{1}{{\sqrt 2 }} \le \frac{{K\ell }}{{2\left( {Ka - Mg} \right)}} < 1$, ce qui exige $Ka > Mg$ et $1 < \frac{{2\left( {Ka - Mg} \right)}}{{K\ell }} \le \sqrt 2 $. Donc${\theta _1}$ existe si $K\left( {a - \frac{\ell }{{\sqrt 2 }}} \right) \le Mg < K\left( {a - \frac{\ell }{2}} \right)$
    Le poids ne doit être, ni trop grand (alors il n’y a qu’une position d’équilibre, qui est stable, en θ = 0), ni trop petit (alors l’action du ressort l’emporte et la position d’équilibre θ = 0 est instable ; lorsqu’on s’en écarte, la perle est rappelée au delà de sa position extrême $\theta = \frac{\pi }{2}$).

  12. L’équilibre est stable si $\frac{{d{F_\theta }}}{{d\theta }} < 0$ et instable si $\frac{{d{F_\theta }}}{{d\theta }} > 0$. Comme :
    $\frac{{d{F_\theta }}}{{d\theta }} = - \left( {Ka - Mg} \right)\cos \theta - \frac{{K\ell }}{2}\cos \frac{\theta }{2}$, la position d’équilibre θ = 0 est stable si $Mg \le K\left( {a - \frac{\ell }{2}} \right)$(alors, c’est la seule position d’équilibre) et instable si l’inégalité est en sens contraire.
    Si θ1 existe, θ = ± θ1 sont des positions d’équilibre stable, parce que les positions d’équilibres sont alternativement stables et instables. On peut aussi le montrer en examinant le signe de $\frac{{d{F_\theta }}}{{d\theta }} = \left( {Ka - Mg} \right)\cos {\theta _1} - \frac{{K\ell }}{2}\cos \frac{{{\theta _1}}}{2}$
    $ = \left( {Ka - Mg} \right){\left\{ {2{{\left[ {\frac{{K\ell }}{{2\left( {Ka - Mg} \right)}}} \right]}^2} - 1} \right\}_1} - \frac{{K\ell }}{2}\frac{{K\ell }}{{2\left( {Ka - Mg} \right)}} = \frac{{{K^2}{\ell ^2}}}{{4\left( {Ka - Mg} \right)}} - \left( {Ka - Mg} \right)$ qui est négatif puisque $\frac{{K\ell }}{{2\left( {Ka - Mg} \right)}} < 1$.
  13. $a = \frac{{2Mg}}{K}$ ; $\ell = \sqrt {3\frac{{Mg}}{K}} $. D’où : $\cos \frac{{{\theta _1}}}{2} = \frac{{\sqrt 3 }}{2}$ ; ${\theta _1} = \frac{\pi }{3}$.
    Les positions d’équilibre sont donc $\theta = - \frac{\pi }{3},\;\theta = 0\;et\;\theta = + \frac{\pi }{3}$.
    Comme $\frac{{d{F_\theta }}}{{d\theta }}\left( {\theta = 0} \right) = \left( {1 - \frac{{\sqrt 3 }}{2}} \right)Mg > 0$, la position d’équilibre θ = 0 est instable.
    Comme $\frac{{d{F_\theta }}}{{d\theta }}\left( {\theta = \pm {\theta _1}} \right) = - \frac{1}{4}Mg < 0$, les positions d’équilibre θ = ± θ1 sont stables.
    L’énoncé complique inutilement la résolution de ce problème en obligeant à intégrer F pour obtenir Ep, puis en dérivant deux fois Ep pour obtenir la dérivée de F. Il est plus simple de ne dériver qu’une fois F.
  14. Si ε est petit, une expression approximative de la force au voisinage de la position d’équilibre est : ${F_\theta } \approx \varepsilon \frac{{d{F_\theta }}}{{d\theta }}\left( {\theta = \pm {\theta _1}} \right) = - \frac{{Mg}}{4}\varepsilon $. La loi fondamentale de la dynamique s’écrit $Ma\ddot \varepsilon = - \frac{{Mg}}{4}\varepsilon $ qui est l’équation d’un oscillateur harmonique de pulsation Ω telle que ${\Omega ^2} = \frac{g}{{4a}} = \frac{K}{{8M}}$ .
  15. Si on résout naïvement l’énoncé, on trouve une amplitude de ε égale à $\sqrt 2 $ radian, ce qui est trop grand.
    L’énergie cinétique initiale est ${E_c}\left( 0 \right) = \frac{1}{2}M{a^2}\dot \varepsilon _0^2 = \frac{{Mga}}{4} = 0,25Mga$.
    La différence d’énergie potentielle entre le point de départ (minimum) et le point extrême possible (maximum) est ${E_p}\left( {\theta = \frac{\pi }{2}} \right) - {E_p}\left( {\theta = \frac{\pi }{3}} \right) = Mga\left( {\frac{5}{2} - \sqrt 6 } \right) \approx 0,05Mga$, qui est nettement plus petit que l’énergie cinétique initiale. Donc la perle a un mouvement approximativement uniforme $\varepsilon \approx \varepsilon \left( 0 \right)t$ jusqu’à ce qu’elle parvienne à l’extrémité du demi cercle. L’énoncé ne permet pas de savoir ce qu’il advient ensuite.

  16. ${\Omega ^2} = \frac{K}{{8M}} = \frac{{K'}}{M}$. Donc $K' = \frac{K}{8} = 125\;N/m$. K’ est nettement plus petit que K, car le poids est la force de rappel la plus importante et diminue notablement l’effet du ressort.
  17. ${\Omega ^2} = \frac{g}{{4a}} = \frac{g}{L}$. Donc $L = 4a = \frac{{8Mg}}{K} = 7,8\;cm$. A noter l’irréalisme de la petitesse de a. En l’absence de ressort, le système est équivalent à un pendule de longueur a, donc on aurait L = a. Le ressort jouant contre le poids, tout se passe comme si la pesanteur était plus faible, ou, ce qui revient au même, comme si la longueur du pendule était plus grande.
  18. ${\vec \sigma _O} = M{a^2}\dot \theta \vec k$.
  19. $O\vec P \wedge F = a{F_\theta }\vec k$.
  20. $\frac{{d{{\vec \sigma }_O}}}{{dt}} = O\vec P \wedge \vec F$ soit $M{a^2}\ddot \theta = a{F_\theta }$ qui au facteur a près est identique à l’équation de la question 9. Il vaut mieux ne pas expliciter la force, la démonstration est alors plus claire.

Concours Physique ENSI (P') 1992 Physique 2 (Corrigé)

Préliminaire.

Soit une machine stationnaire traversée par un écoulement stationnaire d’un fluide et qui reçoit le travail $W'$ et la chaleur $Q$. Appliquons le premier principe au système formé par cette machine et par le fluide qui dans l’état initial est en partie dans la machine et en partie à l’entrée et dans l’état final est en partie dans la machine et en partie à la sortie. Notons par l’indice e le fluide à l’entrée et par l’indice s le fluide à la sortie. Soit ${S_e}$ et ${S_s}$ les sections du tuyau à l’entrée et à la sortie et ${L_e}$ et ${L_s}$ les longueurs qu’y occupe le fluide dans l’état initial et dans l’état final. Outre $W'$, le système reçoit le travail des forces de pression à l’entrée ${p_e}{S_e}{L_e}$, le travail des forces de pression à la sortie $ - {p_s}{S_s}{L_s}$ et le travail du poids qui est $mg({z_e} - {z_s})$ parce que le poids dérive d’une énergie potentielle $mgz$ et parce que la machine et l’écoulement sont stationnaires, ce qui fait que l’énergie potentielle qu’ils stockent est constante.

$\Delta (U + {E_c}) = W' + Q + {p_e}{S_e}{L_e} - {p_s}{S_s}{L_s} + mg({z_e} - {z_s})$
Comme l’écoulement et la machine sont dans un état stationnaire, l’énergie stockée dans la machine ne varie pas, donc $\Delta (U + {E_c}) = [U + {E_c}]_e^s$. D’autre part ${H_e} = {U_e} + {p_e}{S_e}{L_e}$ et ${H_s} = {U_s} + {p_s}{S_s}{L_s}$. En divisant par la masse $m$ de fluide transvasée et en représentant par des minuscules les grandeurs massiques : $[h + \frac{1}{2}{v^2} + gz]_e^s = w' + q$ , soit en négligeant les énergies potentielle et cinétique macroscopiques :${h_s} - {h_e} = w' + q$
1. Compresseur.
1. 1. et 1. 2. La transformation est une adiabatique réversible d’un gaz parfait, qui obéit à la loi de Laplace :
$\begin{array}{l}\frac{{{T_3}}}{{{p_1}^{1 - \frac{1}{\gamma }}}} = \frac{{{T_2}}}{{{p_2}^{1 - \frac{1}{\gamma }}}} \Rightarrow {T_3} = {T_2}{\left( {\frac{{{p_1}}}{{{p_2}}}} \right)^{1 - \frac{1}{\gamma }}} = 273{(12,65/5)^{1 - \frac{1}{{1,2}}}} = 318,7K\\{c_{pg}} = \frac{{\gamma R}}{{(\gamma - 1)M}} = \frac{{1,2 \times 8,31}}{{0,2 \times 0,0865}} = 576,4{\mathop{\rm J}\nolimits} .{K^{ - 1}}.k{g^{ - 1}}\\w' = {h_s} - {h_e} = {c_{pg}}({T_3} - {T_2}) = 576,4 \times (318,7 - 273) = 26300J/kg\end{array}$
1.3. Comme la transformation est adiabatique et réversible, $\Delta s = 0$.
1.4.a. Comme pour un gaz parfait $s - {s_0} = {c_{pg}}\ln \frac{T}{{{T_0}}} - \frac{R}{M}\ln \frac{p}{{{p_0}}}{\rm{ }}$, l’équation de l’isobare de pression $p$ est : $T = {T_0}{\left( {\frac{p}{{{p_0}}}} \right)^{1 - \frac{1}{\gamma }}}\exp \frac{{s - {s_0}}}{{{c_{pg}}}}$
L’isobare de pression ${p_2}$ se déduit de l’isobare de pression ${p_1}$ par translation parallèle à l’axe des $s$ de $ - \frac{R}{M}\ln \frac{{{p_2}}}{{{p_1}}}$ puisque $s(T,{p_2}) = s(T,{p_1}) - \frac{R}{M}\ln \frac{{{p_2}}}{{{p_1}}}$.
1.4.b. Le travail du compresseur est la variation d’enthalpie dans la transformation : $({T_2},{p_2}) \to ({T_3},{p_1})$. D’après la deuxième loi de Joule, $h$ ne dépend pas de $p$ :
$w' = h({T_3},{p_1}) - h({T_2},{p_2}) = h({T_3},p) - h({T_2},p)$
Or, pour une isobare, $dh = Tds \Rightarrow h({T_3},p) - h({T_2},p) = \int\limits_{{T_2}}^{{T_3}} {Tds} $ le long de l’isobare $p$ ; $w'$ est donc l’aire hachurée entre l’isobare $p$ et l’axe des $s$ dans la figure qui suit :

$q < \oint {T\,ds} $ et $w'$ est supérieur à l’aire du cycle.

2. Condenseur.
2.1. ${q_1} = {h_g}({T_1}) - {h_g}({T_3}) + {h_L}({T_1}) - {h_g}({T_1}) = {c_{pg}}({T_1} - {T_3}) - {L_v}({T_1}) = 576,4 \times (305 - 318,7) - 175000 = - 182900{\mathop{\rm J}\nolimits} .k{g^{ - 1}}$
2.2.$\Delta s = {c_{pg}}\ln \frac{{{T_1}}}{{{T_3}}} - \frac{{{L_v}({T_1})}}{{{T_1}}} = 576,4\ln \frac{{305}}{{318,7}} - \frac{{175000}}{{305}} = - 599,1{\mathop{\rm J}\nolimits} .{K^{ - 1}}.k{g^{ - 1}}$
3. Détendeur.
3.1. Cela résulte du premier principe et de l’absence de travail et de chaleur.
3.2. On peut calculer les variations des fonctions d’état sur le chemin : $({\text{liquide à }}{T_1},{p_1}) \to ({\text{liquide à }}{T_2},{p_2}) \to ({\text{fraction }}x{\text{ gazeuse et fraction }}1 - x{\text{ liquide à }}{T_2},{p_2})$
Les propriétés d’un liquide dépendent peu de la pression : on peut calculer sa variation d’enthalpie sans se préoccuper de la pression.
$\begin{array}{l}\Delta h = {c_L}({T_2} - {T_1}) + x{L_v}({T_2}) = 0\\x = \frac{{{c_L}({T_1} - {T_2})}}{{{L_v}({T_2})}} = \frac{{1,38 \times (305 - 273)}}{{205}} = 0,2154\end{array}$
3.3. $\Delta s = {c_L}\ln \frac{{{T_2}}}{{{T_1}}} + \frac{{x{L_v}({T_2})}}{{{T_2}}} = 1380\ln \frac{{273}}{{305}} + \frac{{0,2154 \times 205000}}{{273}} = 8,8{\mathop{\rm J}\nolimits} .{K^{ - 1}}.k{g^{ - 1}}$
4. Evaporateur.
4.1. ${q_2} = (1 - x){L_v}({T_2}) = (1 - 0,2154) \times 205000 = 160800{\mathop{\rm J}\nolimits} .k{g^{ - 1}}$
4.2. $\Delta s = \frac{{(1 - x){L_v}({T_2})}}{{{T_2}}} = 589,1{\mathop{\rm J}\nolimits} .{K^{ - 1}}.k{g^{ - 1}}$
5. L’efficacité est le rapport du gain, c’est-à-dire la chaleur communiquée à l’habitation à chauffer $ - {q_1}$, au coût, qui est l’énergie à donner au compresseur $\frac{{w'}}{r}$. C’est donc $e = r\frac{{ - {q_1}}}{{w'}} = 0,8\frac{{182900}}{{26300}} = 5,56$

On peut améliorer le rendement si on peut rapprocher les températures ${T_1}$ et ${T_2}$. Tel quel, ce chauffage est 5,56 fois moins coûteux en énergie que le chauffage électrique.
6.1. Pour un cycle, le premier principe s’écrit $w' + {q_1} + {q_2} = 0$. Or la même somme calculée avec les valeurs trouvées au fil de ce problème donne $w' + {q_1} + {q_2} = 26300 - 182900 + 160800 = 4200{\mathop{\rm J}\nolimits} .k{g^{ - 1}}$ ; la vérification est acceptable à la rigueur (erreur 2 %).
Les données du problème sont-elles cohérentes ? Elles devraient vérifier $\frac{{d{L_v}}}{{dT}} = \frac{{d{h_g}}}{{dT}} - \frac{{d{h_L}}}{{dT}} = {c_{pg}} - {c_L}$. Or $\frac{{{L_v}({T_1}) - {L_v}({T_2})}}{{{T_1} - {T_2}}} = \frac{{175000 - 205000}}{{305 - 273}} = - 937,5{\mathop{\rm J}\nolimits} .{K^{ - 1}}.k{g^{ - 1}}$ , tandis que ${c_{pg}} - {c_L} = 576,4 - 1380 = - 803,6{\mathop{\rm J}\nolimits} .{K^{ - 1}}.k{g^{ - 1}}$
6.2. Comme l’entropie est une fonction d’état, la somme des variations d’entropie lors du cycle devrait être nulle. Or, elle vaut $\sum {\Delta s} = - 599,1 + 8,8 + 589,1 = - 1,2{\mathop{\rm J}\nolimits} .{K^{ - 1}}.k{g^{ - 1}}$ ; la vérification est acceptable (erreur 0,2 %).
7.1.
$\begin{array}{l}kdT = - ak(T - {T_e})dt\\adt = - \frac{{dT}}{{T - {T_e}}}\\at = - \int\limits_{{T_4}}^{{T_5}} {\frac{{dT}}{{T - {T_e}}}} \Rightarrow a = \frac{1}{t}\ln \frac{{{T_4} - {T_e}}}{{{T_5} - {T_e}}} = \frac{1}{{14400}}\ln \frac{{20}}{{10}} = {4,81.10^{ - 5}}{{\mathop{\rm s}\nolimits} ^{ - 1}}\end{array}$
7.2. ${P_e} = \frac{{ak({T_4} - {T_e})}}{e} = \frac{{{{4,81.10}^{ - 5}} \times {{2.10}^7} \times 20}}{{5,56}} = \frac{{19250}}{{5,56}} = 3460{\mathop{\rm W}\nolimits} $
7.3. La puissance rejetée dans la source froide est ${P_2} = \frac{{160840}}{{182900}}19250 = 16930W$.
Le débit d’eau froide est donc $\frac{{{P_2}}}{{{c_f}\Delta T}} = \frac{{16930}}{{4180 \times 4}} = 1,013{\mathop{\rm kg}\nolimits} .{s^{ - 1}}$.

Concours Physique ENSAM (Options T et TA) 1991 (Énoncé)

Electricité ‑ Optique ‑ Mécanique
(Options T et TA) Durée : 4 h
ELECTRICITE
PREMIERE PARTIE
On considère le circuit de la figure E.1. dans lequel l'interrupteur Tr est fermé depuis un temps suffisamment long pour que le régime permanent soit établi. On s'intéresse au régime transitoire qui suit l'ouverture de l'interrupteur à l'instant t=0.
1.1 Etablir l'équation différentielle concernant vs en exprimant ses coefficients en fonction de L, C, r, R.
1.2 Déterminer numériquement ces coefficients à partir des valeurs numériques suivantes:
E = 15 volts, r = 5 ohms, L = 0,1 henry, C = 1000 microfarads, R = 200 ohms.
1.3 Résoudre cette équation différentielle et exprimer vs = f(t) sous la forme
${v_s} = A - B{\rm{.}}{e^{ - \alpha t}}{\rm{.}}\cos \left( {\omega t + j} \right)$
Représenter sommairement vs = f(t).

DEUXIEME PARTIE
Etude d'un variateur élévateur de tension continue.
On étudie le fonctionnement du dispositif de la figure E.2. destiné à délivrer aux bornes d'une résistance R une tension vs, dont les variations vs devront rester faibles, à partir d'une tension continue positive constante Ve. Le dispositif est construit autour d'un composant électronique Tr commandé par une tension périodique de période T.
On adopte les hypothèses suivantes:
‑ Tr est assimilable à un interrupteur parfait : de 0 à t1 l'interrupteur est fermé et la tension à ses bornes est nulle (u = 0) quel que soit le courant iT qui le traverse; de t1 à T l'interrupteur est ouvert, le courant qui le traverse est nul quelle que soit la tension u à ses bornes.
‑ D est une diode supposée parfaite: vD = 0 quand iD >0 (sens direct) et D = 0 quand vD <0 (sens inverse).
‑ L est une inductance supposée parfaite (résistance négligée).
‑ C est une capacité de forte valeur.
On s'intéresse uniquement au fonctionnement en régime périodique établi.
2.1. Quand l'interrupteur Tr est fermé, quel est l'état de la diode D ? (vs positive).
Que fait la diode D quand l'interrupteur Tr s'ouvre ? Justifier qualitativement votre réponse (préciser le rôle de l'inductance).
On admet que l'état de la diode D reste le même jusqu'à la fin de l'intervalle de temps pendant lequel Tr est ouvert.
2.2. Dans toute la suite du problème on néglige les variations du courant dans la résistance R, c'est-à-dire que ce courant est assimilé à sa valeur moyenne Is. Vs représente la valeur moyenne de la tension vs aux bornes de R et de C.
Justifier cette hypothèse par des relations et des considérations physiques simples concernant les éléments R et C; préciser les valeurs moyennes des courants ic et iD.
2.3. En étudiant successivement les deux états du circuit, montrer qu'en régime périodique établi, i varie entre deux valeurs extrêmes imin et imax. Donner deux expressions de i = imax ‑imin et, de leur égalité déduire le rapport Vs/Ve en fonction du rapport cyclique = t1/T.
2.4 Donner une représentation graphique sommaire de u, i, iD, iT, en fonction du temps, suivant le modèle de la figure E.3.
2.5 Application numérique: on donne Ve = 15 volts, on désire obtenir Vs = 48 volts; la fréquence de fonctionnement de l'interrupteur Tr est f = 20 kHz et L = 0,1 henry.
Préciser la valeur nécessaire de ainsi que de i.
2.6 Exprimer la valeur moyenne de i soit Im et la valeur moyenne de iD soit IDm ; quelle relation lie ces deux valeurs moyennes, exprimer cette relation à l'aide de = t1/T.
2.7 Exprimer la condition correspondant à l'hypothèse faite au 2.1 sur l'état de la diode D quant Tr est ouvert; montrer qu'on en déduit une limite inférieure Lmin de L.
Si R = 200 ohms, préciser numériquement Is, Im et Lmin avec les valeurs numériques déjà indiquées.
2.8 Donner une interprétation énergétique des phénomènes correspondant aux deux parties de la période T.
2.9 En négligeant toujours les variations du courant dans la résistance R, exprimer la quanti d'électricité Q échangée entre la capacité C et le reste du circuit pendant les deux parties de la période T; en déduire la variation vs de la tension vs aux bornes de C et de R.
Avec les valeurs numériques précédentes et C = 1000 microfarads, calculer vs.


OPTIQUE
Franges d'interférences à deux ondes
On propose le dispositif expérimental de la figure O.1. :
Deux miroirs plans M1 et M2, carrés de 4 centimètres de côté, ont un côté commun, leurs faces réfléchissantes sont en regard et leurs plans font entre eux un angle /2 ‑ avec = 2.10-3 radian.
Une source ponctuelle S, émettant une lumière monochromatique, de longueur d'onde = 6.10-7 mètre, est placée sur la droite d'intersection des deux plans de symétrie du dispositif et éclaire les faces réfléchissantes des deux miroirs.
Soit SA = d la distance de la source S au côté commun des deux miroirs;
on donne d = 10 centimètres.
1 ‑ Déterminer la région de l'espace où l'on peut observer des interférences entre :
‑ le faisceau réfléchi par M1, puis par M2
‑ le faisceau réfléchi par M2 , puis par M1
Pour quelle raison faut-il, ici, connaître la dimension des miroirs ?
2 ‑ On reçoit les deux faisceaux réfléchis sur un écran E perpendiculaire, en un point O, à AS et placé à une distance, AO = D, du côté commun des miroirs. On donne D = 1 mètre.
Justifier de l'observation de franges rectilignes sur E et préciser l'orientation de ces franges.
Calculer la largeur, sur l'écran, du système des franges observées, l'interfrange, le nombre de franges brillantes et le nombre de franges noires.

3 ‑ Montrer que l'on ne change pas la netteté des franges en remplaçant la source ponctuelle S par une fente fine parallèle au côté commun des deux miroirs. Montrer que cette netteté diminue si on élargit la fente.
La fente ayant une largeur de 8,25 10-5 mètre, représenter par un graphique les répartitions d'intensité données sur l'écran par le milieu et par chacun des deux bords de la fente. Montrer qu'en fait les franges ont disparu.
4 ‑ La fente étant, à nouveau, très fine, on place sur le trajet des faisceaux réfléchis, perpendiculairement à AS et à 15 cm du côté commun des miroirs, une lentille convergente L de distance focale f = 10 cm (figure O.2.).
Calculer la largeur totale et l'interfrange du nouveau système de franges obtenu sur l'écran E.
5 ‑ Pourquoi, à votre avis, ne vous a-t'on pas fait étudier des interférences qui peuvent être données plus directement par:
‑ le faisceau réfléchi par Ml
‑ le faisceau réfléchi par M2.
MECANIQUE
PREMIERE PARTIE
Deux masses m sont assujetties à se déplacer sur un axe horizontal, x'x, n'introduisant aucun frottement. Les deux masses sont d'une part, reliées entre elles par un ressort et d'autre part, reliées à deux points fixes A et B par deux autres ressorts (figure M.1). Les trois ressorts sont identiques, de masse négligeable et de même raideur k. La distance des points A et B est telle que la tension des trois ressorts est nulle lorsque les deux masses sont immobiles en leur position de repos. On désignera par x1 et x2 les déplacements de chacune des deux masses; x1 et x2 seront contrôlés algébriquement selon l'orientation de x'x précisée sur la figure. On posera o2 = k/m.
1.1 Etablir les équations différentielles qui lient les expressions instantanées de x1 et de x2.
1.2. Le système d'équations différentielles obtenu peut avoir pour solution des oscillations sinusoïdales de même pulsation pour x1 et pour x2 ; en exploitant ce fait, établir une équation donnant les seules pulsations possibles et calculer les valeurs de ces pulsations avec k = 25 N.m-1 et m = 5.10-2 kg.
1.3. Pour chaque pulsation précédente, quelle relation lie les expressions instantanées de x1 et x2 ? Préciser physiquement le mouvement des deux masses.
Comment lancer les deux masses à l'instant initial de leur mouvement pour obtenir chacune des oscillations sinusoïdales communes ?; Justifier physiquement de l'expression des pulsations obtenues.

DEUXIEME PARTIE
On considère maintenant le dispositif de la figure M.2.
Deux pendules simples identiques, de longueur l et de masse m, peuvent se mouvoir dans un même plan vertical autour de deux axes parallèles situés dans le même plan horizontal. Les masses m ont été réunies par un ressort de masse négligeable et de raideur k. La tension du ressort est nulle lorsque les pendules sont verticaux.
Dans tout le problème, on ne considérera que des mouvements de petite amplitude: le ressort reste horizontal et on peut alors confondre le déplacement des extrémités du ressort avec les composantes horizontales x1 et x2 des déplacements de chacune des masses mobiles par rapport à sa position d'équilibre. On contrôlera algébriquement x1 et x2 selon l'orientation de l'axe x'x précisée sur la figure. On notera g l'accélération de la pesanteur. On négligera tout phénomène de frottement.
2.1. Etablir, à nouveau, le système d'équations différentielles qui lient les expressions instantanées de x1 et x2.
2.2. Procéder à la même recherche que celle faite dans la première partie quant à l'existence de solutions sinusoïdales de même pulsation pour x1 et pour x2.
On posera: $\omega {{}_1^2} = \frac{{kl + mg}}{{ml}}$ et $\gamma = \frac{{kl}}{{kl + mg}}$
Exprimer les pulsations propres aux oscillations sinusoïdales communes de x1 et x2 en fonction de 1 et .
Calculer leurs valeurs avec l = 1 mètre et g = 9,80 mètre.seconde-2, m et k ayant les valeurs données à la première question.
2.3. Donner pour chaque pulsation précédente, les liaisons entre les expressions de x1 et x2 et commenter physiquement les mouvements.

Concours Physique Centrale-Supélec (M, P') 1991 Physique II (Corrigé)

Corrigé centrale 91 M-P'
Première partie.
I- Collision neutron-noyau
1/ Conservation de la qdm : $m{\vec V_1} = m{\vec V_2} + M{\vec w_2} \Rightarrow {\vec V_1} = {\vec V_2} + A{\vec w_2}$
Conservation de l'énergie: $\text{ }\!\!{\scriptscriptstyle 1\!/\!{ }_2}\!\!\text{ }m\vec{V}_{1}^{2}=\text{ }\!\!{\scriptscriptstyle 1\!/\!{ }_2}\!\!\text{ }m\vec{V}_{2}^{2}+\text{ }\!\!{\scriptscriptstyle 1\!/\!{ }_2}\!\!\text{ }M\vec{w}_{2}^{2}\Rightarrow \vec{V}_{1}^{2}=\vec{V}_{2}^{2}+A\vec{w}_{2}^{2}$
2/ De ${\vec V_1} = {\vec V_2} + A{\vec w_2}$, on tire : $\vec V_2^2 = {({\vec V_1} - A{\vec w_2})^2} = \vec V_1^2 + A\vec w_2^2 - 2A{V_1}{w_2}\cos \theta $
Soit $\cos \theta = \frac{{\vec V_1^2 - \vec V_2^2 + {A^2}\vec w_2^2}}{{2A{V_1}{w_2}}} = \frac{{A\vec w_2^2 + {A^2}\vec w_2^2}}{{2A{V_1}{w_2}}} = \frac{{{w_2}}}{{{V_1}}}\frac{{1 + A}}{2}$> 0 donc 0 < θ < π/2
En fonction des énergies : $\text{ }\!\!{\scriptscriptstyle 1\!/\!{ }_2}\!\!\text{ }m\vec{V}_{1}^{2}=\text{ }\!\!{\scriptscriptstyle 1\!/\!{ }_2}\!\!\text{ }m\vec{V}_{2}^{2}+\text{ }\!\!{\scriptscriptstyle 1\!/\!{ }_2}\!\!\text{ }M\vec{w}_{2}^{2}\Rightarrow {{E}_{1}}-{{E}_{2}}=\text{ }\!\!{\scriptscriptstyle 1\!/\!{ }_2}\!\!\text{ A}\,\text{m}\,\vec{w}_{2}^{2}$ et ${{E}_{1}}=\text{ }\!\!{\scriptscriptstyle 1\!/\!{ }_2}\!\!\text{ }\,\text{m}\,\vec{V}_{1}^{2}$
Alors $\cos \theta = \frac{{{w_2}}}{{{V_1}}}\frac{{1 + A}}{2} = \sqrt {\frac{{{E_1} - {E_2}}}{{A{E_1}}}} \frac{{1 + A}}{2}$donc $\frac{{{E_2}}}{{{E_1}}} = 1 - \frac{{4A{{\cos }^2}\theta }}{{{{(1 + A)}^2}}}$

II- Modèle des sphères dures.
1/ La force de contact passe par le centre d'inertie, donc la vitesse ${\vec w_2}$ sera dirigé suivant la réaction normale. On en déduit : $\sin \theta = \frac{b}{{{R_1} + {R_2}}}$
2/ Le paramètre d'impact peut varier entre 0 et la valeur R1 + R2. Ce qui correspond pour le centre du neutron à
à une cible de surface variant de 0 à (R1 + R2)2.
La probablité de recevoir un impact sur une couronne de rayon : b → b + db est :$\frac{{dP}}{1} = \frac{{2\pi bdb}}{{\pi {{({R_1} + {R_2})}^2}}}$
3/ Par définition: $ < - Ln\,[1 - K{\cos ^2}\theta ]{ > _b} = < - Ln\,[1 - \frac{{K{b^2}}}{{{{({R_1} + {R_2})}^2}}}]{ > _b} = - \int\limits_0^{{R_1} + {R_2}} {Ln[1 - \frac{{K{b^2}}}{{{{({R_1} + {R_2})}^2}}}]\;db} $
En posant $x = \frac{{K{b^2}}}{{{{({R_1} + {R_2})}^2}}}$⇒ $\frac{1}{K}\left[ {(1 - x)Ln(1 - x) - (1 - x)} \right]_0^K = \frac{1}{K}\left[ {(1 - K)Ln(1 - K) - (1 - K) + 1} \right]$
Ce qui donne : $1 + \frac{{1 - K}}{K}Ln(1 - K)$ cqfd . Il faut que 0 < K < 1 pour que la fonction aît un sens.
4/ On a obtenu $\frac{{{E_2}}}{{{E_1}}} = 1 - \frac{{4A{{\cos }^2}\theta }}{{{{(1 + A)}^2}}} = 1 - K{\cos ^2}\theta $ avec $K = \frac{{4A}}{{{{(1 + A)}^2}}}$< 1 si A > 1
on peut utiliser le résultat précédent : $K = \frac{{4A}}{{{{(1 + A)}^2}}} \Rightarrow 1 - K = {\left( {\frac{{A - 1}}{{A + 1}}} \right)^2}$
Donc coefficient de ralentissement : $\gamma = < - Ln\,[\frac{{{E_2}}}{{{E_1}}}]{ > _b} = 1 + {\left( {\frac{{1 - A}}{{\sqrt {2A} }}} \right)^2}Ln(\frac{{A - 1}}{{A + 1}}) = $
5/ a)La dérivée de γ vaut zéro pour : $0 = \left( {\frac{{1 - A}}{{\sqrt A }}} \right)\left\{ { - \left( {\frac{{{A^{1/2}} + {A^{ - 1/2}}}}{{2\sqrt 2 \;A}}} \right)Ln(\frac{{A - 1}}{{A + 1}}) - \left( {\frac{1}{{\sqrt A }}} \right)\left( {\frac{1}{{(A + 1)}}} \right)} \right\}$
Le terme entre crochet ne s'annulant pas, la racine est A = 1. On vérifiera que c'est bien un maximum pour le ralentissement.
b) A-N : 1H (A = 1) γ = 1 ; 2H (A = 2) γ = 0,725 ; 12C (A = 12) γ = 0,158 ; 238U (A = 238) γ = 0,008 ;
III- Application aux ralentissements des neutrons.
1/ Il y a ½ kT par degré de liberté, donc E300K = 3/2kT = 3,9.10−2 eV.
C'est très faible devant l'énergie initiale des neutrons. On peut considèrer les noyaux immobiles, sauf pour les dernières collisions.
2 a/ Avec $\gamma = < - Ln\,[\frac{{{E_2}}}{{{E_1}}}]{ > _b}$ et en écrivant : $\frac{{{E_n}}}{{{E_0}}} = \frac{{{E_n}}}{{{E_{n - 1}}}}\frac{{{E_{n - 1}}}}{{{E_{n - 2}}}}\; \cdots \frac{{{E_1}}}{{{E_0}}} \Rightarrow Ln\left( {\frac{{{E_n}}}{{{E_0}}}} \right) = \sum\limits_1^n {Ln\left( {\frac{{{E_p}}}{{{E_{p - 1}}}}} \right)} $
on a en raisonnant sur les valeurs moyennes : $Ln\left( {\frac{{{E_n}}}{{{E_0}}}} \right) = - n\gamma \Rightarrow {E_n} = {E_0}{e^{ - \gamma }}$
2b/ $n = - \frac{1}{\gamma }Ln\left( {\frac{{{E_{300K}}}}{{{E_0}}}} \right)$d'où 1H : n = 17 ; 2H : n = 24 ; 12C : n = 108 ; 238U : n = 214;
3a/ A une date t : $v(t) = \sqrt {\frac{{2E(t)}}{m}} $, la durée moyenne intercollision est: $\Delta t = \frac{\lambda }{{v(t)}}$et le nombre de collisions par unité de temps est : $\frac{{dn}}{{dt}} = \frac{1}{{\Delta t}} \Rightarrow \frac{{dn}}{{dt}} = \frac{1}{\lambda }\sqrt {\frac{{2E}}{m}} $.
3b/ L'équation $Ln\left( {\frac{{{E_n}}}{{{E_0}}}} \right) = - n\gamma $donne, en passant à la limite : $\gamma \,dn = - Ln\,[\frac{{E + dE}}{E}] = - \frac{{dE}}{E}$
soit : $\gamma \frac{{dt}}{\lambda }\sqrt {\frac{{2E}}{m}} = - \frac{{dE}}{E}$ ; en posant $\,y = \frac{E}{{\;{E_0}}}$ on a $\gamma \frac{{dt}}{\lambda }\sqrt {\frac{{2{E_0}}}{m}} = - \frac{{dy}}{{\;{y^{3/2}}}}$
3c/ L'intégration conduit à : $2\left[ {{y^{ - 1/2}} - 1} \right] = \frac{\gamma }{\lambda }t\,\sqrt {\frac{{2{E_0}}}{m}} $soit : $\,\sqrt {\frac{{{E_0}}}{E}} = 1 + \frac{\gamma }{{2\lambda }}t\,\sqrt {\frac{{2{E_0}}}{m}} $
4a/ On calcule d'abord $\,\sqrt {\frac{{{E_0}}}{E}} \approx 5000$ puis avec γ = 0,158 on trouve t = 120 µs .
On a toujours : $\,\sqrt {\frac{{{E_0}}}{E}} > > 1$ donc $\,t = \frac{{2\lambda }}{\gamma }\sqrt {\frac{m}{{2E}}} $ indépendant de E0.
4b/ La distance parcourue pendant dt est : $dx = v.dt = dt\sqrt {\frac{{2E}}{m}} $ et on a aussi $\gamma \frac{{dt}}{\lambda }\sqrt {\frac{{2E}}{m}} = - \frac{{dE}}{E}$
donc $dx = - \frac{\lambda }{\gamma }\frac{{dE}}{E} \Rightarrow x = \frac{\lambda }{\gamma }Ln\,{\frac{{{E_0}}}{E}_{300K}}$ on trouve ainsi x = 2,8 m.
On peut remarquer que cette distance corespond à nλ puisque $n = - \frac{1}{\gamma }Ln\left( {\frac{{{E_{300K}}}}{{{E_0}}}} \right)$.

Deuxième partie.
1a/ Avec $\xi \,\vec u = {A_1}M \to $ ⇒ le théorème d'Ampère donne$\vec B = \frac{{{\mu _0}I}}{{2\pi {\xi ^2}}}\vec k \wedge \xi \vec u$
1b/${A_1}M \to = $$(r - a\cos \theta ){\vec u_r} + a\sin \theta {\vec u_{^\theta }}$⇒$\vec B = \frac{{{B_0}}}{{{\xi ^2}}}\left\{ \begin{array}{l} - a\sin \theta \;{{\vec u}_r}\\(r - a\cos \theta \;){{\vec u}_\theta }\end{array} \right.$et${\xi ^2} = {a^2} + {r^2} - 2\,a\,r\cos \theta $
1c/ $\vec B' = {B_0}\left\{ \begin{array}{l} - \frac{{a\sin \theta }}{{{\xi ^2}}}\; = - \left[ {\sin \theta {\rm{ + 2}}u\sin \theta \;\cos \theta - {{\rm{u}}^{\rm{2}}}\sin \theta [1 - 4{{\cos }^2}\theta {\rm{]}}} \right]{\rm{ }}\\\frac{{(r - a\cos \theta \;)}}{{{\xi ^2}}} = \left[ {u - {\rm{cos}}\theta - 2{\rm{u}}\,{\rm{co}}{{\rm{s}}^{\rm{2}}}\theta + {{\rm{u}}^{\rm{2}}}\cos \theta [3 - 4{{\cos }^2}\theta ]} \right]\end{array} \right.$
2a/ Il faut faire une rotation de π et changer le signe du courant. Soit: $\vec{B}''(u,\theta )=-\vec{B}'(u,\theta +\pi )$
2b/ ${B_{1r}} = B{'_r}(u,\theta ) - B{'_r}(u,\theta + \pi ) = - 2{B_0}\left[ {\sin \theta - {{\rm{u}}^{\rm{2}}}\sin \theta [1 - 4{{\cos }^2}\theta {\rm{]}}} \right]$
${B_{1\theta }} = B{'_\theta }(u,\theta ) - B{'_\theta }(u,\theta + \pi ) = - 2{B_0}\left[ {{\rm{cos}}\theta - {{\rm{u}}^{\rm{2}}}\cos \theta [3 - 4{{\cos }^2}\theta ]} \right]$
en linéarisant : ${B_{1r}} = - 2{B_0}\left[ {\sin \theta + {{\rm{u}}^{\rm{2}}}\sin 3\theta } \right]$ et${B_{1\theta }} = - 2{B_0}\left[ {{\rm{cos}}\theta + {{\rm{u}}^{\rm{2}}}\cos 3\theta } \right]$
3a/ Il faut faire une rotation d'angle − 2π/3 et d'angle +2π/3 .
3b/ Donc ${B_r} = {B_{1r}}(u,\theta ) + {B_{1r}}(u,\theta - 2\pi /3) + {B_{1r}}(u,\theta + 2\pi /3)$
${B_\theta } = {B_{1\theta }}(u,\theta ) + {B_{1\theta }}(u,\theta - 2\pi /3) + {B_{1\theta }}(u,\theta + 2\pi /3)$
Or $\left\{ \begin{array}{l}\cos (\theta - 2\pi /3) + \cos (\theta + 2\pi /3) = - \cos \theta \\\sin (\theta - 2\pi /3) + \sin (\theta + 2\pi /3) = - \sin \theta \end{array} \right.$on a finalement:
${B_r} = - 2{B_0}\left[ {3{{\rm{u}}^{\rm{2}}}\sin 3\theta } \right]$
${B_\theta } = - 2{B_0}\left[ {3{{\rm{u}}^{\rm{2}}}\cos 3\theta } \right]$ donc $C = 6$
4a/ Ligne de champ: $d\vec \ell //\vec B \Rightarrow \frac{{dr}}{{rd\theta }} = \frac{{{B_r}}}{{{B_\theta }}}$ ⇒$\frac{{dr}}{r} = \frac{{\sin 3\theta }}{{\cos 3\theta }}d\theta \Rightarrow \,{r^3} = r_0^3/\cos 3\theta $
4b/ ci-contre : allure des lignes de champ.
4c/ Module $B(r) = 6{B_0}\;{r^2}/{a^2}$,
lignes isomodules B(r) = Cte sur un cercle de centre O
II- Action du champ sur un neutron
1a/ Pour un dipôle donc deux cas possibles : ${{E}_{//}}=-\,B$ et ${{E}_{\bot }}=\,B$
Soit en remplaçant B par $C{B_0}\;{r^2}/{a^2}$⇒ ${{E}_{//}}=-\text{ }\!\!{\scriptscriptstyle 1\!/\!{ }_2}\!\!\text{ }m{{\Omega }^{2}}{{r}^{2}}$ et ${{E}_{}}=\text{ }\!\!{\scriptscriptstyle 1\!/\!{ }_2}\!\!\text{ }m{{\Omega }^{2}}{{r}^{2}}$
1b/ La force est donnée par : $\vec F = \, - gr\vec ad\,{E_p}$ donc ${\vec F_{//}} = m{\Omega ^2}\,\vec r$ et ${\vec F_{\rlap{--} \rlap{--} \not /\rlap{--} /}} = - m{\Omega ^2}\,\vec r$
Pour confiner il faut une force de rappel, seuls les neutrons antiparallèles peuvent être confinés.
2a/ La RFD donne : ${\vec F_{\rlap{--} \rlap{--} \not /\rlap{--} /}} = - m{\Omega ^2}\,\vec r = m\frac{{{d^2}\vec r}}{{d{t^2}}} + m\frac{{{d^2}z}}{{d{t^2}}}\vec k$ ⇒$ - m{\Omega ^2}\,\vec r = m\frac{{{d^2}\vec r}}{{d{t^2}}}{\rm{ et }}\frac{{{d^2}z}}{{d{t^2}}} = 0$
2b/ L'intégration donne :$\,\vec r(t) = {\vec A_1}\cos \,\Omega t + {\vec A_2}\sin \Omega t$ où ${\vec A_1}{\rm{ et }}{\vec A_2}$ sont des constantes.
soit avec les conditions initiales: $z = {v_0}t$ et $\,\vec r(t) = {x_0}\vec i\cos \,\Omega t + \frac{{{u_0}}}{\Omega }\vec j\sin \Omega t$.
2c/ La trajectoire est une hélice d'axe Oz et de section elliptique.
3a/ Le neutron est confiné si le grand axe de l'ellipse est inférieur au rayon a; x0 étant plus petit que a il faut que:$a > \frac{{{u_0}}}{\Omega }$ soit encore :${u_C} = a\,\Omega $.
3b/ A-N: uC = 5,9 m.s−1ce qui donne EC = 18.10−8 eV et aussi TC = 1,4.10−3 K
Ce résultat justifie l'appellation neutron ultra-froids.
3c/ La fonction de répartition de Boltzmann permet de calculer la fraction de neutrons qui ont une énergie inférieure à la valeur calculée précédemment:
$F = \int\limits_0^{{E_C}} {\frac{1}{{\sqrt {2\pi } }}\frac{1}{{{{(kT)}^{3/2}}}}\sqrt E \exp ( - E/kT)\,dE} $
si T = 300 K << TC on peut simplifier ⇒$F \approx \int\limits_0^{{E_C}} {\frac{1}{{\sqrt {2\pi } }}\frac{1}{{{{(kT)}^{3/2}}}}\sqrt E \,dE} = \frac{1}{{\sqrt {2\pi } }}\frac{1}{{{{(kT)}^{3/2}}}}\frac{2}{3}\left[ {{E^{3/2}}} \right]_0^{{E_C}}$
Soit finalement : $F = \sqrt {\frac{3}{{4\pi }}} {\left[ {\frac{{{T_C}}}{T}} \right]^{3/2}} \approx {5.10^{ - 9}}$ donc extrémement faible.
4/ Les neutrons ont un mouvement de dérive suivant l'axe Oz. or les fils créant le champ magnétique ne peuvent être rééllement infinis. Le confinement n'a lieu que dans la partie centrale du dispositif et se termine lorsque les neutrons sortent du dispositif.
III- Amélioration du confinement

1a/ Pour les neutrons confinés : ${\vec F_{//}} = - m{\Omega ^2}\,\vec r$ avec maintenant $\vec r = $$O'M \to $$ = (\rho - R){\vec u_\rho } + z\vec k$
1b/ En cylindriques : $\vec a = (\ddot \rho - \rho {\dot \theta ^2}){\vec u_\rho } + (2\dot \rho \dot \theta + \rho \ddot \theta ){\vec u_\theta } + \ddot z\vec k$
1c/ Equations du mouvement : $\left\{ \begin{array}{l}\ddot \rho - \rho {{\dot \theta }^2} = - {\Omega ^2}(\rho - R)\\2\dot \rho \dot \theta + \rho \ddot \theta = 0\\\ddot z = - {\Omega ^2}z\end{array} \right.$
2a/ Compte tenu des conditions initiales: $\ddot z = - {\Omega ^2}z \Rightarrow z = {z_0}\cos (\Omega t) + \frac{{{V_0}}}{\Omega }\sin \Omega t$.
2b/ $2\dot \rho \dot \theta + \rho \ddot \theta = \frac{1}{\rho }\frac{{d({\rho ^2}\dot \theta )}}{{dt}} = 0 \Rightarrow {\rho ^2}\dot \theta = Cte = \rho _0^2{\omega _0}$ "mouvement projeté sur x0y à force centrale".
2c/ Il reste l'équation en ρ(t): $\ddot \rho - \rho {\dot \theta ^2} = \ddot \rho - \left( {\frac{{\rho _0^4\omega _0^2}}{{{\rho ^3}}}} \right) = - {\Omega ^2}(\rho - R)$
3a/ si ω0 = 0 alors θ = θ0 est constant : $\ddot \rho = - {\Omega ^2}(\rho - R) \Rightarrow (\rho - R) = ({\rho _0} - R)\cos \Omega t$,
c'est l'équation paramètrique (z(t),ρ(t)) d'une ellipse de centre O'.
3b/ si $\dot \theta = Cte = {\omega _0}$ alors ${\rho ^2} = \rho _0^2$, la trajectoire est sinusoïde dessinée sur un cylindre d'axe Oz.
La trajectoire sera fermée si la durée d'un tour est un multiple de la période, soit $\Omega = n{\omega _0}$.
4a/ Si $\rho = {\rho _m}[1 + \varepsilon (t)]$ alors l'équation en ε est :${\rho _m}\ddot \varepsilon - \left( {\frac{{\rho _0^4\omega _0^2}}{{\rho _m^3}}} \right)[1 - 3\varepsilon ] = - {\Omega ^2}({\rho _m} - R + {\rho _m}\varepsilon )$
4b/ La valeur moyenne correspond à ε = 0 : $ - \left( {\frac{{\rho _0^4\omega _0^2}}{{\rho _m^3}}} \right) = - {\Omega ^2}({\rho _m} - R)$on a
4c/ Par différence : ${\rho _m}\ddot \varepsilon + 3\left( {\frac{{\rho _0^4\omega _0^2}}{{\rho _m^3}}} \right)\varepsilon + {\Omega ^2}{\rho _m}\varepsilon = 0$ soit : $\ddot \varepsilon + 3\left( {\frac{{\rho _0^4\omega _0^2}}{{\rho _m^4}}} \right)\varepsilon + {\Omega ^2}\varepsilon = 0$
ce qui s'intègre en $\varepsilon (t) = {\varepsilon _0}\cos (\Omega 't + {\varphi _0})$ en posant : $\Omega ' = \sqrt {3\left( {\frac{{\rho _0^4\omega _0^2}}{{\rho _m^4}}} \right) + {\Omega ^2}} $.
Ce qui donne alors la vitesse angulaire: $\dot \theta = \frac{{\rho _0^2{\omega _0}}}{{{\rho ^2}}} \approx \frac{{\rho _0^2{\omega _0}}}{{\rho _m^2}}[1 - 2\varepsilon ]$.
4d/ Les trajectoires sont alors ses oscillations autour des sinusoïdes tracées sur un cylindre. La vitesse angulaire étant elle même oscillante.
5/ La pesanteur entaîne un mouvement de chute selon l'équation z = ½ gt2 qui s'ajoute aux oscillations. Au bout d'une période la "chute" vaut donc : $h = 2g{\pi ^2}/{\Omega ^2}$ .
On calcule alors : h = 5,6 mm, ce qui n'est pas négligeable.
___________________________

Autres Concours

2011  : Concours ENAC de  physique 2011  :  énoncé ,  corrigé Concours ICNA de  physique 2011  :  énoncé ,  corrigé Concours ICNA de ...